Mardi received an inheritance of $40,000. She invested part at 9% and the rest at 12%. Her total annual income from the investments was $3900. Find the amount she invested at 9%

Answers

Answer 1

Answer:

She invested $30,000 at 9%

Step-by-step explanation:

Let the amount invested per percentage be $x and $y respectively

Total investment is 40,000

This means that;

x + y = 40,000 ••••••••••(i)

Now for her income;

9/100 * x = 9x/100

for the 12% = 12/100 * y = 12y/100

Now;

9x/100 + 12y/100 = 3,900

9x + 12y = 390000 •••••••••(ii)

we solve both simultaneously

From i, y = 40,000-x

Substitute this into ii

9x + 12(40,000-x) = 390,000

9x + 480,000-12x = 390,000

3x = 480,000-390,0000

3x = 90,000

x = 90,000/3

x = $30,000


Related Questions

How many digits will be in the quotient?
39 4,641

Answers

Answer:

the answer is 119 so three digits

Step-by-step explanation:

Answer:

3 digits

Step-by-step explanation:

4641/39=119

The lateral area of a right cone which has a base diameter of 4 units and a height of 10 units is:

Answers

Answer:

≈64.08

Step-by-step explanation:

Lateral Area=πr[tex]\sqrt{h^{2} +r^{2} }[/tex]

Radius is 1/2 of the diameter so r=2

Height=10

Substitute:

: [tex]3.141(2)\sqrt{10^{2}+2^{2} } \\6.282\sqrt{100+4} \\6.282\sqrt{104} \\ 64.066.282*10.2\\ 64.076\\\\[/tex]

64.076 rounded = 64.08

You are going to use an incline plane to lift a heavy object to the top of a shelving unit with a height of 5 ft. The base of the incline plane is 15 ft from the shelving unit. What is the length of the incline​ plane?

Answers

Answer:

15.8 ft

Step-by-step explanation:

The inclined plane, the base of and the shelving unit form the shape of a right angled triangle.

The hypotenuse is the length of the inclined plane, h.

The base of the triangle is 15 ft.

The height of the triangle is 5 ft.

To find the hypotenuse, h, we have to use Pythagoras rule:

[tex]h^2 = a^2 + b^2[/tex]

where a = height of the triangle

b = base of the triangle

Therefore:

[tex]h^2 = 5^2 + 15^2\\\\h^2 = 25 + 225 = 250\\\\h = \sqrt{250}\\ \\h = 15.8 ft[/tex]

The inclined plane is 15.8 ft long.

Identify the like terms in the expression.

Answers

Answer:D

The answer is D.

Step-by-step explanation:

Like terms are only related to the variable at the end, and the variable also has to have the same exponent or it is not a like term.

En un programa de televisión se hacen 30 preguntas. Por cada respuesta correcta se suman 8 puntos, por cada respuesta errónea se restan 5 puntos y por aquellas preguntas que no se contesten no se suman ni se restan puntos. Si un participantes obtuvo 13 puntos, ¿Cuantas respuestas erróneas pudo tener?

Answers

Answer:

7 wrong answers.

Step-by-step explanation:

We know that when you answer a question well, you earn 8 points and a bad one loses 5 points, this means that when you answer 1 good and 1 bad, there is a total of 3 points (8-5).

This means that when answering 5 good and 5 bad, in total it would be 3 * 5 points, that is to say 15 points, that is, it goes through 2 points. To subtract 2 points, you would have to answer 1 good and 2 bad, (8 - 2 * 5), which turns out to be -2 points.

In total there would be 6 questions right and 7 questions wrong, like this:

8 * 6 + 5 * 7 = 13

13 points, in 13 questions (6 correct + 7 incorrect)

The rest of the questions were not answered so that the score does not go up or down.

Which means you got 7 wrong answers.

John has two jobs and earns a total of $2,345 per month. What percent of his gross income does John receive from his second job where he makes $609.70 a month?

Answers

Answer:

10000000.9

Step-by-step explanation:


Before graduating this year, a senior homeroom was given a survey. Of those surveyed, 24% felt they learned better at home. Of this group, 80% said they plan on taking an online course in college. Of the students who felt they did not learn better at home, 40% said they plan on taking an online course in college

Part A
What is the probability a person who does not plan on taking an online course felt they learned better at home?

A : 2/21
B : 24/125
C : 38/125
D : 19/31
E :None of these

Part B
What is the probability a person who does plan on taking an online course felt they did not learn better at home?

A : 2/21
B : 24/125
C : 38/125
D : 19/31
E : None of these

Answers

Answer:

(A) The correct option is (A).

(B) The correct option is (E).

Step-by-step explanation:

The events can be defined as follows:

X =  students felt they learned better at home

Y = students plan on taking an online course in college

The information provided is:

P (X) = 0.24

P (Y|X) = 0.80

P (Y|X') = 0.40

[tex]P(Y'|X)=1-P(Y|X)\\=1-0.80\\=0.20[/tex]

[tex]P(Y'|X')=1-P(Y|X')\\=1-0.40\\=0.60[/tex]

The Bayes' theorem states that the conditional probability of an event E[tex]_{i}[/tex] given that another event A has already occurred is:

[tex]P(E_{i}|A)=\frac{P(A|E_{i})P(E_{i})}{\sum {P(A|E_{i})P(E_{i})}}[/tex]

(A)

Compute the probability a person who does not plan on taking an online course felt they learned better at home as follows:

Use the Bayes' theorem.

[tex]P(X|Y')=\frac{P(Y'|X)P(X)}{P(Y'|X)P(X)+P(Y'|X')P(X')}[/tex]

              [tex]=\frac{0.20\times 0.24}{(0.20\times 0.24)+(0.60\times 0.76)}\\\\=0.09524\\\\\approx 0.095[/tex]

Thus, the probability a person who does not plan on taking an online course felt they learned better at home is 0.095 or 2/21.

(B)

Compute the probability a person who does plan on taking an online course felt they did not learn better at home as follows:

[tex]P(X'|Y')=1-P(X|Y')\\=1-0.095\\=0.905[/tex]

Thus, the probability a person who does plan on taking an online course felt they did not learn better at home is 0.905.

k (t) = 10t - 19
k(-7) =

Answers

Answer:

k(-7) = - 89

Step-by-step explanation:

k (t) = 10t - 19

Let t = -7

k(-7) =10 * -7 - 19

        =-70-19

        -89

Answer:

[tex]k (-7) = -89[/tex]

Step-by-step explanation:

[tex]k (t) = 10t - 19[/tex]

[tex]k (-7) = 10(-7) - 19[/tex]

[tex]k (-7) = -70 - 19[/tex]

[tex]k (-7) = -89[/tex]

Please answer correctly !!!! Will mark brainliest !!!!!!!!!!!!

Answers

Answer:

[tex]x^2+10x+24[/tex]

Step-by-step explanation:

So just multiply the sides:

[tex]x^2+6x+4x+24[/tex]

Which is:

[tex]x^2+10x+24[/tex]

please solve y = 3x - 1​

Answers

Step-by-step explanation:

i think question is not complete.

What is the slope of the graph? slope = -1/3 slope = -3 slope = 3 slope = 1/3

Answers

Answer:

The slope is -3

Step-by-step explanation:

This is so because the line in pointing down, leading to the fact that the slope is negative, and rise over run is 3/1......

Therefore, the slope of the line is -3

To find the slope of the line, first start with a point on the graph.

Let's use the point (0,3) which we call point A.

The other point we will call point B.

Now remember that slope can be found using the ratio rise/run

between any two points that are on that line.

To get from point A to point B along this line, we must

first go down 3 units so we say that our rise is -3.

From there we move 1 unit to the right so our run is 1.

So our slope or rise/run is -3/1 which reduces to -3.

Brass contains copper and zinc in the ratio 2:1. How much ZINC is there in 750 g of brass?

Answers

Answer:

250

Step-by-step explanation:

copper: zinc : total

2              1      2+1 =3

We have 750 brass

750/3 = 250

Multiply each by 250

copper: zinc : total

2*250     250  3*250

500        250       750

pleass more math help

Answers

Can’t answer this question

Jasper decided to save $100 at the end of each month for a year and deposit it in a bank account that earns an annual interest rate of 0.3%, compounded monthly. Use the formula for an annuity, F, to determine how much money will be in the account at the end of the 6th month, rounding your answer to the nearest penny.
Note: Your interest rate must be converted to a decimal

Answers

Answer: 600.38

Step-by-step explanation:

just put in the answer you lazy

8. Where will the hour hand of a clock stop if it starts:
a.
from 7 and turns through 1 right angle?
b. from 11 and turns through 3 right angles

can you plz say me the answer​

Answers

Answer:

a. 11

b. 9

Step-by-step explanation:

thats the answer

What the answer to this

Answers

Answer:

I think it might be a I don't know for sure though I just need some more points so I can ask a question myself sorry if this didn't help

Please help me!!!!!!

Answers

Answer:

C

Step-by-step explanation:

The addition of the x^5 term makes it non proportional

Answer:

C.

Step-by-step explanation:

A. y = (6x + 3) - 3= 6x

B. y = - 15x

D. = - 1/3 x

All of these are proportional because they have general formula y = kx.

The price of a computer was decreased by 7% to £500. What was the price before the decrease? Give your answer to the nearest penny.

Answers

Answer:

£537.63

Step-by-step explanation:

Let the original price of the computer be x.

The price of a computer was decreased by 7% to £500. This implies that:

x - 7/100 * x = 500

x - 0.07x = 500

0.93x = 500

x = 500/0.93 = £537.63

The price before the decrease was £537.63.

Can someone please help

Answers

Answer:

The numbers to input are 2, 2 , 7

Step-by-step explanation:

252 = 2² * 3² * 7

So the numbers to input are 2, 2 , 7

(6) Work out
5 1
6 12

Answers

Answer:

=123

Step-by-step explanation:

HELPPPPPP ITS ABOUT EQUATIONSSSS HELPPPPP Explanation needed HELPPPPP PLEASEE THIS IS THE LAST QUESTION LEFTTTTTTT

Answers

Answer:

C. the average total cost for the first month of a gym membership

Step-by-step explanation:

x=1 is for month 1,

the value of y includes 1-month fee and one off payment, so this is the average total for the first month of membership

Answer:

It is the average total cost for the first month of a gym membership

Step-by-step explanation:

y = 34.99x+49

The 49 is the cost to join the gym and the 34.99 is the monthly cost

Let x =1 which is the cost after one month

It includes the cost to join and the 1st month membership

It is the average total cost for the first month of a gym membership

Find the value of a . A.18 B.21 C.20 D.17

Answers

Answer:

a =18

Step-by-step explanation:

The two angles are vertical angles and vertical angles are equal

6a +11 = 2a+83

Subtract 2a from each side

6a-2a +11 = 2a-2a+83

4a +11 =83

Subtract 11 from each side

4a +11 -11 = 83-11

4a = 72

divide each side by 4

4a/4 = 72/4

a =18

The circumference of the earth is given. 


Circumference of earth: 24,901 miles


What is the diameter of earth? Round your answer to the nearest tenth. Use 3.14 for π.


Answers

Answer:

7930.3 miles = d

Step-by-step explanation:

The circumference equals

C = pi *d

24901 = 3.14 d

Divide each side by 3.14

24901 / 3.14 = d

7930.254777 = d

Rounding to the nearest tenth

7930.3 =d

Please answer correctly !!!!!!! Will mark brainliest !!!!!!!!!!

Answers

Answer:

9 years.

Step-by-step explanation:

Make x the number of years passed.

[tex]\frac{39+x}{3+x}=4[/tex]

12 + 4x = 39 + x

27 = 3x

x = 9

The line plot shows the result of a survey asking students how many hours they spent reading last week. How many students spent 5 or more hours reading?

Answers

Answer:

15 total students

Step-by-step explanation:

Based on the line plot picture that is attached below it can be calculated that a total of 15 students spent 5 or more hours last week reading. This can be easily calculated by adding all the x's on the marks of 5 hours or more. With both the boys and girl students combined the tally was the following

5 hours : 4 students

6 hours : 1 student

7 hours : 3 students

8 hours : 3 students

9 hours : 0 students

10 hours: 2 students

11 hours : 2 students

12 hours : 0 students

adding up to 15 total students

A man divides his 360 cattle between his son in the ratio 7: 6: 5. Find the smallest share​

Answers

360/18=20

20*7=140

20*6=120

20*5=100

Answer=140:120:100

Answer: smallest share is 100

Step-by-step explanation:

7:6:5

Add the numbers so 7+6+5=18

Divide 360 by 18 which equals 20

Multiply each ratio by 20 so...

7×20:6×20:5×20

140:120:100

Which angle in ADEF has the largest measure?

Answers

Answer:

F is the largest angle

Step-by-step explanation:

The largest angle is opposite the largest side.  The smallest angle is opposite the smallest side.

The largest side is 4 so the largest angle is F

PLS HELP

Simplify the function f(x) =
3x
(81)
4
Then determine the key aspects of the function.
The initial value is
The simplified base is
The domain is
The range is

Answers

Answer:

Step-by-step explanation:

This follows the form

[tex]y=a(b)^x[/tex]

where a is the initial value and b is the base with the exponent.  Using that information, we can see that the initial value in our function is 1/3. Simplifying the base will take some work. Let's first rewrite this is a radical:

[tex]81^{\frac{3x}{4} }=\sqrt[4]{81^{3x} }[/tex]

Now let's break up 81 into its factors. 81 is 9*9 which is 3*3*3*3.  Therefore,

[tex]81=3^4[/tex]

We will use that as a simplification:

[tex]\sqrt[4]{(3^4)^{3x}}[/tex]  which simplifies to

[tex]\sqrt[4]{3^{12x}}[/tex]

Rewriting that as an exponent looks like this:

[tex]3^{\frac{12x}{4}}[/tex] which simplifies to

[tex]3^{3x}[/tex]

That's the answer for the second part.  The whole exponential equation now is

[tex]f(x)=\frac{1}{3}(3)^{3x}[/tex]

The domain for an exponential function is all real numbers and the range is

y > 0

Answer:

1/3

27

all real numbers

y > 0

ON EDGE

Step-by-step explanation:

(3x5−2x4−5)−(2x4+x2−10) Subtract the two polynomials

Answers

Answer:

3x^5-4x^4-x^2+5

Step-by-step explanation:

(3x^5−2x^4−5)−(2x^4+x^2−10)

Distribute the minus sign

(3x^5−2x^4−5)−2x^4-x^2+10

Combine like terms

3x^5-4x^4-x^2+5

Hello!

Answer:

[tex]\boxed{ \bf 3x^5~-~4x^4~-~x^2~+~5}[/tex]

__________________________________Explanation:

(3[tex]x^{5}[/tex] - 2[tex]x^{4}[/tex] - 5) - (2[tex]x^{4}[/tex] + x² - 10)

Drop the brackets:

3[tex]x^{5}[/tex] - 2[tex]x^{4}[/tex] - 5 - 2[tex]x^{4}[/tex] - x² + 10

Combine Like Terms:

3[tex]x^{5}[/tex] - 2[tex]x^{4}[/tex] - 2[tex]x^{4}[/tex] - x² - 5 + 10

3[tex]x^{5}[/tex] - 4[tex]x^{4}[/tex] - x² + 5

What is the shape of the cross-section formed when a cylinder intersects a
plane as shown in the drawing?

Answers

Answer:

circle,

Step-by-step explanation:

disc in the middle vertically in this case is equal to the bases

The intersection is called an Oval. Hence the correct option is A. An oval in mathematics is a shape just like a circle but with an elongated outline like the shape of an egg.

What is a Cross-section?

A cross-section is a surface, an area that is created or exposed by executing a straight cut across or through a shape.

Cross-sections in technical drawings are used for depicting the internal view of an object that is three-dimensional.

Learn more about Cross-Sections are:

https://brainly.com/question/10511133

Other Questions
Case Studv Donna and Darren have three children. Darren's father has hypercholesterolemia, which is transmitted in an autosomal-dominant pattern. Darren gets his cholesterol levels checked frequently, and they have remained within normal limits; however, two of Donna and Darren's three children have high plasma levels of total and low-density lipoprotein (LDL) cholesterol. Darren does not understand why his children are affected when it is clear that he is healthy Draw the family pedigree for this family What could explain why Darren is unaffected but two of his children are affected? Darren return from a meeting with the genetic counselor but still does not understand what she told him. How could you help Darren understand what is going on with his family? how much is $1000 in Great Britain-pounds() PLEASE HELP ME! Which statement is NOT true about direct proportion?A. direct proportion is a type of relationship between two variablesB. when one variable doubles, the other quadruplesC. when one variable doubles, the other doubles tooD. when one variable is zero, the other is also zero What do scientists think the effect will be of having more greenhouse gases in our atmosphere?A. More warming of the atmosphere B. More cooling of the atmosphere C. Less air in the atmosphere D. More air in the atmosphere During the Civil War, President Lincoln shifted his goals concerning slaveryfrom:A. tolerating slavery only in the North to accepting it in all states.B. accepting slavery in the South to freeing all Southern slaves.C. demanding immediate emancipation to accepting a gradual end toslaveryD. opposing all forms of slavery to allowing slavery in the borderstates. In the ordered pair (3, 4), the value of y is 4. The 4 tells us how many places from zero we should go up or down. Will you go up or down from zero? One of the results of the chaos created by the Vietnam war was that a neighboringcountry of Cambodia fell to the violent communist known as what?Khmer RougeLung DwoCommunist CompanionsRed Rebels A probability model for rolling a cube with faces numbered 1 through 6 is: Please Help, Math Problem..... Using the given function, select the correct set of ordered pairs for the following domain values.{-12, -3, 0, 3, 12}OA. {(-12, -18), (-3, -3), (0,2), (3,7), (12,22)}OB. {-18, -12), (-3, -3), (2,0), (7,3), (22, 12)oc. {-12, -3), (-3, -3), (0, -3), (3, 3), (12, 6)}OD. {{-43, -12), (-3, -3), 4,0), (0, 3), (6,12)} Please answer correctly !!!!!!! Will mark brainliest !!!!!!!!! Please help with corrections Help quickly pleaseee Japan Tsunami some Interesting facts solve for X will give brainliest!! the little boxes on the bottom with the numbers in it are the answers. plz do all three if u can 3) Cassandra earns $56,000 this year. She gets a cost of living raise to compensate for yearly inflation of 2%. What will her raise be next year?Enter your answer as a number, for example: 4210 Mr. Drummonds class has 24 students. Monday there were 3 students absent. What percent of the class were absent? (1 point) Which scatter plot suggests a linear relationship between x and y?A) I onlyB) I and IV onlyC) I, II and IV onlyD) I,III, and IV only Maris Brothers Inc. needs a cash disbursement schedule for the months of April, May, and June following information in its preparation.Sales:February = $483,000;March $525,000;April $542,000;May $629,000;June $657,000;July $667,000Purchase: Purchases are calculated as 55% of the next month's sales, 10% of purchases are made in cash, 45% of purchase are paid for 1 month after purchase and the remaining 45% of purchases are paid for 2 months after purchase.Rent: The firm pays rent of $8,030 per monthWages and salaries: Base wage and salary cost are fixed at $5,800 per month plus a variable cost of 6.8% of the current month's sales.Taxes: A tax payment of $ $54,100 is due in JuneFixed asset outlays: New equipment costing $74,000 will be bought and paid for in April.Interest payments: An interest payment of $30,400 is due in June.Cash dividends: Dividends of $12,500 will be paid in AprilPrincipal repayments and retirements: No principal repayments or retirements are due during these months.